LSAT and Law School Admissions Forum

Get expert LSAT preparation and law school admissions advice from PowerScore Test Preparation.

 Administrator
PowerScore Staff
  • PowerScore Staff
  • Posts: 8916
  • Joined: Feb 02, 2011
|
#41315
Complete Question Explanation
(The complete setup for this game can be found here: lsat/viewtopic.php?t=15716)

The correct answer choice is (D)

The condition in the question stem asserts that both an N and a G film will be shown, and that information in combination with the super-rule creates the following relationship:
J06_Game_#1_#7_diagram 1.png

Because of the size of the block behind N, the first N cannot be shown in any of the last three slots. From the Not Laws, we also know that N cannot be shown on day 2, and hence the only days remaining for the first N to be shown are day 1 and day 3. Therefore, answer choice (D) is correct.
You do not have the required permissions to view the files attached to this post.
 melissa27
  • Posts: 38
  • Joined: Jan 17, 2012
|
#3469
The answer listed for this question is (D) day 1 or else day 3.

But how would the rule fit into day 1?

If N>GIN, and N went into day 1, N would also have to go into day 4 which would violate the Super rule, wouldn't it?


refer pg (3-44)- lesson 3 hw
User avatar
 Dave Killoran
PowerScore Staff
  • PowerScore Staff
  • Posts: 5853
  • Joined: Mar 25, 2011
|
#3474
Hi Melissa,

Thanks for the question. As you note, the correct answer is indeed (D). Let's take a closer look at why that is the case.

First, you are right that we have a N > GIN super-rule in play in this question. But, when considering whether N could be shown on day 1, it looks like you interpreted it it as a NGIN block (forcing N to be shown on day 4, which would be a violation). But, that doesn't have to be the case since the "GIN" portion is a separate piece from the initial N. Thus, N could be first, but then the GIN block could be in a position other than 2-3-4, avoiding the violation you mention.

Please let me know if that makes sense. Thanks!
 melissa27
  • Posts: 38
  • Joined: Jan 17, 2012
|
#3494
Yes absolutely, thank you so much!
 Blueballoon5%
  • Posts: 156
  • Joined: Jul 13, 2015
|
#40734
I am confused with the phrasing of this question. From the answer key in the back of the homework section (in the book), it seems like the question was only referring to the first Norwegian film (in the following chain: N > GIN). How come the question is ignoring the placement of the second Norwegian film?

Because, if the first Norwegian film is placed in the first section, then it is possible for the second Norwegian to be in the fifth day. So, that would make 5th day possible (answer choice E), right?
User avatar
 Dave Killoran
PowerScore Staff
  • PowerScore Staff
  • Posts: 5853
  • Joined: Mar 25, 2011
|
#40789
Hi Blue,

The key to this question is seeing that it is a Must Be True question, so while N can be shown on day 5, it's not the case that N must be shown on day 5, which figures into these answers. For example, the following hypothetical is possible—N-F-H-G-I-N—which shows that N doesn't have to be on day 3 or else day 5.

In this problem, answer choices (A), (B), and (C) are relatively easy to work with, but (D) and (E) are much trickier because they pose scenarios where N is claimed to be limited to at least one of the pair of days. So, if we can show one day doesn't have to occur, that's not enough to eliminate these answers; we have to show that both days are out. Our hypothetical does that for (E), knocking it out.

Please let me know if that helps. Thanks!
 Blueballoon5%
  • Posts: 156
  • Joined: Jul 13, 2015
|
#41013
Thanks Dave! Yes, I understand it now!! :)
 Naj
  • Posts: 8
  • Joined: Dec 16, 2019
|
#72920
Question #7: Local, Must Be True. The correct answer choice is (D)
The condition in the question stem asserts that both an N and a G film will be shown, and that
information in combination with the super-rule creates the following relationship:
N G I N
Because of the size of the block behind N, the first N cannot be shown in any of the last three
slots. From the Not Laws, we also know that N cannot be shown on day 2, and hence the only days
remaining for the first N to be shown are day 1 and day 3. Therefore, answer choice (D) is correct.

I do not understand, if we are considering the size of the block behind N, then how is it possible that N can be in day 1 what about the block behind it (G,I)

https://forum.powerscore.com/lsat/viewtopic.php?t=1658 [admin note: question and reply moved to this post]
 Adam Tyson
PowerScore Staff
  • PowerScore Staff
  • Posts: 5153
  • Joined: Apr 14, 2011
|
#72921
There is some additional discussion of this question in this thread, Naj:

https://forum.powerscore.com/lsat/viewtopic.php?t=1658 [admin note: question and reply moved to this post]

The short answer here is that there is an N somewhere before a GIN block. They do not have to be adjacent - the first N could be in the first space, and the GIN block could occupy slots 4, 5, and 6, for example. That setup alone proves that there does not have to be an N in slot 5, eliminating answer C.

To clarify, when we say the GIN block is behind N, we mean it comes after N in the order, like this:

N :longline: GIN

That first N must be either in slot 1 or else slot 3, and that's answer D.

I hope that helps! If not, try drawing it out and see if you can come up with a solution where there is no N in either slot 1 or slot 3, but still complying with all the rules. I think you'll find you cannot do so, proving that an N must be in one of those slots.
 Iceberg.Human
  • Posts: 4
  • Joined: Mar 04, 2020
|
#75926
Hi,I've been wondering how I can know that it's only THE FIRST N that this question asks here given the way the question is rephrased.(The question is " If a film in Greek is going to be shown at some time after a film in Norwegian, then a film in Norwegian must be shown on" and nothing about whether or not it's only the first or including both mentioned, while explanation says "hence the only days remaining for the first N to be shown are day 1 and day 3". )
Or, is it the case that regardless of how the explanation is rephrased, D is correct because it is only D that can be "Must be true' among all the answer choices here? (and my question above is not something that one should particularly care...?)

Could you help me with this? Thank you!
Last edited by Iceberg.Human on Fri Jun 05, 2020 2:26 am, edited 1 time in total.

Get the most out of your LSAT Prep Plus subscription.

Analyze and track your performance with our Testing and Analytics Package.